0

Problem: Let $x, y > 0$. Prove that $$f(x, y) = 2(x + y)\ln \frac{x + y}{2} - (x + 1)\ln x - (y + 1)\ln y \ge 0.$$

$\phantom{2}$

My attempt:

For $a > 0$, we need to prove that $$f\left(x\right)=(x+a)\ln\frac{a+x}{2}-\frac{\left(a+1\right)\ln a+\left(x+1\right)\ln x}{2} \ge 0.$$

WLOG assume that $a\leq x$

The second derivative can be rewrite as :

$$a(1-x)+x(x+1)\geq 0$$

Wich is obvious with the contraint above .

We deduce that the derivative admits a single zero and is increasing .



Edit to clarify the problem :

It's a straightforward consequence of my answer here show this inequality $\sqrt{\frac{a^b}{b}}+\sqrt{\frac{b^a}{a}}\ge 2$ in the final version .

Well after MartinR comment :I need to show it to conclude my answer .






Other idea Case $0<a\leq 1$ and $x\geq a$:

Using simple bound got from https://www.researchgate.net/publication/267163352_Proofs_of_three_open_inequalities_with_power-exponential_functions lemma 7.2 where $b=0$ and $d=\frac{x+a}{2}$ we have the new function :

$$h(x)=2\left(\frac{\left(x+a\right)}{2}-1\right)-0.5\left(\left(x+1\right)\ln\left(x\right)+\left(a+1\right)\ln\left(a\right)\right)$$

It's NOT true that $h(x)\geq 0$ in this case . See the remarks below

Some remarks :

The derivative does not depends from $a$ .

the abscissa of the minimum of $f(x)$ denoted by $x_{min}$ is less or equal than the $x_0$ where $h(x_0)=0$ for $0<a\leq 1$ and $x\geq a$

Important remark :

It seems we have for $0<a\leq 1$ and $x\geq a$:

$$x_{min}\leq 2-a+0.5\left(a-1\right)^{2}\leq x_{0}\quad (I)$$

It reduces the problem to a single variable inequality in $a$ .

Some Conjectures :

Firstly It seems we have $a\in (0,1]$:

$$0\leq f'\left(2-a+0.5\left(a-1\right)^{2}\right)$$

Secondly It seems we have $a\in (0,1]$ :

$$0\leq h\left(2-a+0.5\left(a-1\right)^{2}\right)$$



Question :

How to prove it ?

Thanks.

River Li
  • 49,125
Barackouda
  • 3,879

3 Answers3

4

Problem: Let $x, y > 0$. Prove that $$f(x, y) = 2(x + y)\ln \frac{x + y}{2} - (x + 1)\ln x - (y + 1)\ln y \ge 0.$$

Proof:

Note that $f(1, 1) = 0$. We need to prove that $x = y = 1$ is the global minimizer of $f(x, y)$ on $x,y > 0$.

We have \begin{align*} f(x, y) &\ge 2x \ln \frac{x}{2} + 2y\ln \frac{y}{2} - (x + 1)\ln x - (y + 1)\ln y\\ &= (x - 1)\ln x - 2x\ln 2 + (y - 1)\ln y - 2y\ln 2. \tag{1} \end{align*}

Fact 1: $(u - 1)\ln u - 2u\ln 2 > -3$ for all $u > 0$.
(The proof is given at the end.)

Fact 2: $(u - 1)\ln u - 2u\ln 2 > 4$ for all $u\in (0, 1/64]\cup [16, \infty)$.
(The proof is given at the end.)

By (1) and Facts 1-2, if $x < 1/64$ or $x > 16$, then $f(x, y) > -3 + 4 = 1$, and if $y < 1/64$ or $y > 16$, then $f(x, y) > -3 + 4 = 1$.

Since $f(1, 1) = 0$, the minimum of $f(x, y)$ on $x, y > 0$ occurs on the region $1/64 \le x, y \le 16$.

The minimum of $f(x, y)$ on the region $1/64 \le x, y \le 16$ may occur in the interior of the region (stationary points), or it may occur on the boundary of the region.

First, we can prove that $x = y = 1$ is the only stationary point of $f(x, y)$ on $x, y > 0$:

Fact 3: If $x, y > 0$ satisfies $\frac{\partial f}{\partial x} = \frac{\partial f}{\partial y} = 0$, then $x = y = 1$.
(The proof is given at the end.)

Second, at the boundaries of the region $1/64 \le x, y \le 16$, it is easy to verify that $f(x, y) > 1$ using (1) and Facts 1-2.

As a result, $f(x, y) \ge 0$ for all $1/64 \le x, y \le 16$.

Thus, $f(x, y) \ge 0$ for all $x, y > 0$.

We are done.



Proof of Fact 1:

Let $h(u) = (u - 1)\ln u - 2u\ln 2 + 3$. Since $(u - 1)\ln u \ge 0$ for all $u > 0$, we have $h(u) > 0$ for all $0 < u < \frac{3}{2\ln 2}$. If $u \ge \frac{3}{2\ln 2}$, using $\ln 2 < \frac{4}{5}$ and $\ln z \ge \frac{2(z - 1)}{z + 1}$ for all $z \ge 1 $, we have $$h(u) \ge (u - 1)\cdot \frac{2(u - 1)}{u + 1} - 2u\cdot \frac{4}{5} + 3 = \frac{2u^2 - 13u + 25}{5u + 5} > 0.$$

We are done.


Proof of Fact 2:

Let $H(u) = (u - 1)\ln u - 2u\ln 2 - 4$. We have $H'(u) = \ln u + 1 - \frac{1}{u} - 2\ln 2$.

If $0 < u \le \frac{1}{64}$, we have $H'(u) < 0$ and thus $H(u) \ge H(1/64) = \frac{47}{8}\ln 2 - 4 > 0$.

If $u \ge 16$, we have $H'(u) > 0$ and thus $H(u) \ge H(16) = 28\ln 2 - 4 > 0$.

We are done.


Proof of Fact 3:

We have \begin{align*} \frac{\partial f}{\partial x} &= 2\ln\frac{x + y}{2} + 1 - \ln x - \frac{1}{x}, \tag{2}\\ \frac{\partial f}{\partial y} &= 2\ln\frac{x + y}{2} + 1 - \ln y - \frac{1}{y}. \tag{3} \end{align*}

(2) gives $$y = -x + 2\sqrt{\mathrm{e}^{-1}x\mathrm{e}^{1/x}}. \tag{4}$$

[(2) - (3)] gives $$\ln x + \frac{1}{x} = \ln y + \frac{1}{y}. \tag{5}$$

From (4) and (5), we have $$\ln x + \frac{1}{x} - \ln(-x + 2\sqrt{\mathrm{e}^{-1}x\mathrm{e}^{1/x}}) - \frac{1}{-x + 2\sqrt{\mathrm{e}^{-1}x\mathrm{e}^{1/x}}} = 0.$$

From $-x + 2\sqrt{\mathrm{e}^{-1}x\mathrm{e}^{1/x}} > 0$, we have $\frac{1}{x}\mathrm{e}^{1/x} > \mathrm{e}/4$ which results in $0 < x < x_0$ where $x_0$ is the unique real root of $\frac{1}{x}\mathrm{e}^{1/x} = \mathrm{e}/4$.

Let $$g(x) = \ln x + \frac{1}{x} - \ln(-x + 2\sqrt{\mathrm{e}^{-1}x\mathrm{e}^{1/x}}) - \frac{1}{-x + 2\sqrt{\mathrm{e}^{-1}x\mathrm{e}^{1/x}}}.$$ We have $$g'(x) = - \frac{\sqrt{\mathrm{e}^{-1}x\mathrm{e}^{1/x}}}{x^2(-x + 2\sqrt{\mathrm{e}^{-1}x\mathrm{e}^{1/x}})^2}\cdot h(x) - \frac{(x - 1)^2\sqrt{\mathrm{e}^{-1}x\mathrm{e}^{1/x}}}{x^2(-x + 2\sqrt{\mathrm{e}^{-1}x\mathrm{e}^{1/x}})^2} $$ where \begin{align*} h(x) &= 2\sqrt{x}\,(1 - x)(\mathrm{e}^{-1/2 + 1/(2x)} - 1) + \sqrt{x}\,(1 - \sqrt{x})^2\\ &\quad + 2x\sqrt{x}\left[\mathrm{e}^{1/2-1/(2x)} - 1 - \left(\frac{1}{2} - \frac{1}{2x}\right)\right]. \end{align*}

Since $(1 - x)(\mathrm{e}^{-1/2 + 1/(2x)} - 1) \ge 0$ for all $x > 0$, and $\mathrm{e}^{1/2-1/(2x)} - 1 - \left(\frac{1}{2} - \frac{1}{2x}\right) \ge 0$ for all $x > 0$, we have $h(x) \ge 0$ for all $x > 0$.

Thus, $g'(x) < 0$ for all $x \in (0, 1)\cup (1, x_0)$. Also, $g'(1) = 0$ and $g(1) = 0$. Thus, $g(x) = 0$ has exactly one real root, say $x = 1$.

From (4), we have $y = 1$.

We are done.

River Li
  • 49,125
  • @MartinR The remaining is to check $f$ at the boundaries. Actually, you can find $a, b$ such that the minimum of $f$ occurs on $a \le x, y \le b$. So, you check the boundaries, and the stationary point. – River Li Oct 20 '21 at 10:24
  • @MartinR I found a material to help you to understand my comment. https://collegeparktutors.com/blog/finding-absolute-maximums-and-minimums-of-a-2-variable-function-on-a-closed-region – River Li Oct 20 '21 at 10:47
  • @MartinR You are right. Thanks. – River Li Oct 20 '21 at 11:11
  • Can you tell me if my conjecture are correct in the OP ? Is it hard ? Can you prove it and add it in your proof ? thannnnkkkss! – Barackouda Oct 22 '21 at 18:00
  • @ErikSatie They are both true. I will not add this irrelevant thing in my proof. – River Li Oct 23 '21 at 02:14
  • Congrats River ! A last question can we generalised it to the Simplifire's question ? https://math.stackexchange.com/questions/4278217/show-that-left-fracx-1x-2x-2-rightp-left-fracx-2x-3x-3-righ?noredirect=1&lq=1 – Barackouda Oct 25 '21 at 12:06
  • @ErikSatie I don't know. – River Li Oct 25 '21 at 12:11
  • I have my own proof now what do you think about ? – Barackouda Oct 26 '21 at 12:35
  • @ErikSatie You should write down clearly the detailed complete proof. – River Li Oct 26 '21 at 13:39
  • @ErikSatie You should make your proof easy to read. For example, last time, your write $k''' < 0$, but I did not see where $k$ is defined. Later I found you wrote $m\left(a\right)+m\left(2-a+0.5\left(a-1\right)^{2}\right)=k(a)=\cdots$. I think it is not nice to give the definition of $k$ in this place. I think you should write: Let $k(a) = \cdots$... We have $k''' < 0$. – River Li Oct 26 '21 at 13:55
  • @ErikSatie By the way, do you mean to prove the OP, or just the two conjectures? – River Li Oct 26 '21 at 14:01
  • both see my edits :-) . – Barackouda Oct 30 '21 at 11:21
  • @ErikSatie Do you prove the OP? – River Li Oct 30 '21 at 11:28
  • Yes because for $a\leq x$ the function is convex and admitts a single minimum wich I show to be greater than zero using Vasc's lemma 7.2.If you have some question I'm here . – Barackouda Oct 30 '21 at 11:33
  • @ErikSatie Can you elaborate on that as an answer (giving a complete detailed proof, not just two conjectures)? – River Li Oct 30 '21 at 12:50
  • @ErikSatie By the way, it is not true that $h(x) \ge 0$ (using Vasc's lemma 7.2)? – River Li Oct 30 '21 at 12:52
  • It's true on a particular interval wich is sufficient to show the inequality . – Barackouda Oct 30 '21 at 17:47
  • @ErikSatie If so, just write down the complete step-by-step proof clearly. – River Li Oct 31 '21 at 00:14
  • @ErikSatie I did not downvote your another answer for another topic. But I think your writing is hard to read. You should make your expression nice. For example, in $6\left(\frac{\left(a+x+c\right)}{3}-1\right)$, parentheses are not necessary, $6\left(\frac{a+x+c}{3}-1\right)$ is ok; in $\left(0.6a^{0.8}+c^{-0.2}\right)\ln\left(a\right)$, I think $\ln a$ is better than $\ln (a)$ (see Vasc's article). You should edit your expression manually. – River Li Oct 31 '21 at 00:37
2

Remarks: This is to give an alternative proof of Fact 3 in my answer.
The new proof is related to the following three questions:
Estimate the bound of the sum of the roots of $1/x+\ln x=a$ where $a>1$,
and
https://artofproblemsolving.com/community/c6h2849102,
and
Lower bound for the square root sum of the roots of $x - \ln x - m$.


Problem: Let $x, y > 0$. Prove that $$f(x, y) = 2(x + y)\ln \frac{x + y}{2} - (x + 1)\ln x - (y + 1)\ln y \ge 0.$$

Fact 3: If $x, y > 0$ satisfies $\frac{\partial f}{\partial x} = \frac{\partial f}{\partial y} = 0$, then $x = y = 1$.

Proof of Fact 3:

We give the following auxiliary result.

Lemma 1: Let $a > 1$. The equation $\frac{1}{z} + \ln z = a$ has two distinct real roots $z_1 < z_2$. Then $2\ln \frac{z_1 + z_2}{2} + 1 > a$. (The proof is given at the end.)

We have \begin{align*} \frac{\partial f}{\partial x} &= 2\ln\frac{x + y}{2} + 1 - \ln x - \frac{1}{x}, \\ \frac{\partial f}{\partial y} &= 2\ln\frac{x + y}{2} + 1 - \ln y - \frac{1}{y}. \end{align*}

From $\frac{\partial f}{\partial x} = \frac{\partial f}{\partial y} = 0$, we have $$2\ln\frac{x + y}{2} + 1 = \ln x + \frac{1}{x} = \ln y + \frac{1}{y}. \tag{1}$$

If $x = y$, from (1), we have $x = y = 1$ (easy).

If $x \ne y$, from $\ln x + \frac{1}{x} = \ln y + \frac{1}{y}$, we have $\ln x + \frac{1}{x} > 1$ (easy). However, by Lemma 1, $2\ln\frac{x + y}{2} + 1 = \ln x + \frac{1}{x}$ is impossible.

As a result, we have $x = y = 1$.

We are done.


Proof of Lemma 1:

We use the parametrization by wyz@AoPS. Let $t = z_2/z_1 > 1$. We have \begin{align*} \frac{1}{z_1} + \ln z_1 &= a,\\ \frac{1}{tz_1} + \ln(t z_1) &= a \end{align*} which results in $\frac{1}{z_1} = \frac{1}{tz_1} + \ln t$ or $z_1 = \frac{t - 1}{t\ln t}$. Then we have $z_2 = \frac{t-1}{\ln t}$ and $a = \ln \frac{t - 1}{\ln t} + \frac{\ln t}{t - 1}$.

It suffices to prove that, for all $t > 1$, $$2\ln \frac{\frac{t - 1}{t\ln t} + \frac{t-1}{\ln t}}{2} + 1 > \ln \frac{t - 1}{\ln t} + \frac{\ln t}{t - 1}$$ or $$h(t) := 2\ln\frac{1+t}{2t} + \ln \frac{t - 1}{\ln t} + 1 - \frac{\ln t}{t - 1} > 0.$$ We have \begin{align*} h'(t) &= \frac{(t^2 + t)\ln^2 t + (t - 1)^3\ln t - (t + 1)(t - 1)^2}{t(1 + t)(t - 1)^2\ln t}\\[6pt] &> \frac{(t^2 + t)\cdot \frac{2(t - 1)}{t + 1}\cdot\ln t + (t - 1)^3\ln t - (t + 1)(t - 1)^2}{t(1 + t)(t - 1)^2\ln t} \tag{2}\\[6pt] &= \frac{t^2 + 1}{2t(t^2 - 1)\ln t}\left(\ln (t^2) - \frac{2(t^2 - 1)}{t^2 + 1}\right)\\[6pt] &> 0 \tag{3} \end{align*} where we have used $\ln t > \frac{2(t-1)}{t + 1}$ for all $t > 1$ in (2) and (3). Also, we have $\lim_{t\to 1^{+}} h(t) = 0$. Thus, we have $h(t) > 0$ for all $t > 1$.

We are done.

River Li
  • 49,125
  • @RiverLi Thanks that nice I have also a second proof can you show for $x,y\in(0,\infty)$ and $x\neq y$ : $$\left(x+1\right)\ln\left(x\right)<\left(x+y\right)\ln\left(\frac{x+y}{2}\right)+\frac{\left(1+x+y\right)}{x+2y}\left(x-y\right)$$ ? – Barackouda Feb 12 '23 at 12:18
  • @ErikSatie I have no idea. – River Li Feb 12 '23 at 12:39
  • @RiverLi Woopsie it's false ...Try : $$\left(x+1\right)\ln x<\left(x+y\right)\ln\left(\frac{x+y}{2}\right)+\left(x-y\right)+\frac{\left(x-y\right)^{3}}{12\left(x+y\right)^{2}}$$ for $x,y>0,x\neq y$ . Remains to add the two inequalities . – Barackouda Feb 12 '23 at 12:58
  • @ErikSatie I believe it is true. Very nice. It is some kind of isolate fudging. If you can prove it nicely, then your proof is very nice. – River Li Feb 12 '23 at 13:05
-1



First conjecture :

Define :

$$F(x)=f'(x)$$

We have $a\in(0,1)$ :

$$F(2-a+0.5\left(a-1\right)^{2})=\ln\left(a+2-a+0.5\left(a-1\right)^{2}\right)-\frac{0.5}{2-a+0.5\left(a-1\right)^{2}}-0.5\ln\left(2-a+0.5\left(a-1\right)^{2}\right)+0.5-\ln\left(2\right)$$

Using differentiation we have :

$$(F(2-a+0.5\left(a-1\right)^{2}))'=\frac{\left(a^{5}-10a^{4}+40a^{3}-74a^{2}+63a-20\right)}{\left(a^{2}-4a+5\right)^{2}\left(a^{2}-2a+5\right)}=\frac{\left(a-1\right)^{2}\left(a^{3}-8a^{2}+23a-20\right)}{\left(a^{2}-4a+5\right)^{2}\left(a^{2}-2a+5\right)}<0$$

We are done !




Second conjecture :

We need to show for $x\in(0,1]$ :

$$p(x)=(x-1)^2-(x+1)\ln(x)-(2-x+0.5(x-1)^2+1)\ln(2-x+0.5(x-1)^2)\geq 0$$

The second derivative is :

$$p''(x)=\frac{-\left(-2x^{2}+8x-6\right)}{\left(x^{2}-4x+5\right)^{2}}+\frac{\left(1-x\right)}{x^{2}}+\frac{2}{\left(x-4\right)x+5}-\ln\left(\left(0.5x-2\right)x+2.5\right)-1$$

Now we use the bound $z\in[1,3]$:

$$k(x)=\frac{0.5\left(z^{2}-1\right)}{z}\geq \ln(z)$$

Remains to show :

$$\frac{-\left(-2x^{2}+8x-6\right)}{\left(x^{2}-4x+5\right)^{2}}+\frac{\left(1-x\right)}{x^{2}}+\frac{2}{\left(x-4\right)x+5}-k\left(\left(0.5x-2\right)x+2.5\right)-1\geq 0$$

Wich is smooth using a computer .

So now as conclusion we have showed that the minimum is greater than zero because the abscissa of the minimum of $f(x)$ is less or equal than the root of $h(x)$ .

Barackouda
  • 3,879
  • @RiverLi can you check my progress ? Thanks in advance ! – Barackouda Oct 21 '21 at 15:53
  • See my comment on the OP. Please edit or delete this part in the OP (for example, emphasize $h(x)\ge 0$ is not true for $0 < a \le 1, x\ge 1$): Other idea Case 0<a≤1 and x≥1 "...lemma 7.2 where ... we have the new function..." – River Li Oct 22 '21 at 05:31
  • You can use \tag{something} to label the equation. For example, \tag{V} for your first equation: $0\leq h\left(\frac{1}{1+\frac{\left(x-1\right)\left(x+1\right)}{2}}\right) \tag{V}$ – River Li Oct 22 '21 at 05:39
  • @RiverLi Thanks for the help :-)! – Barackouda Oct 22 '21 at 10:34
  • Why did you say you "I show the hardest part I mean"? If so, you do have a complete proof? I think that the part you does not prove is your hardest part. – River Li Oct 22 '21 at 10:58
  • You said "We have $x \in (0,25, 1]$ ...", do you mean you have proved it? Or you just want to prove it? Do you say "we have" unless you have proved it. – River Li Oct 22 '21 at 11:03
  • In the OP, $f(x, a)$ is symmetric, you may assume $x \ge a$. However, you discuss two cases (not necessary): (1) $0 < a \le 1$, $x \ge 1$; (2) $a \ge 1$ and $0 < x \le 1$. You may edit it. – River Li Oct 22 '21 at 11:28
  • By the way, I did not downvote you. – River Li Oct 22 '21 at 13:48
  • @RiverLi Again thanks ! – Barackouda Oct 22 '21 at 13:56
  • You just comment here. – River Li Oct 22 '21 at 14:12
  • In the 2nd line, you can not say "We have" before you prove it. You should make clear what you have proved. Also, you should separate different parts using
    etc. This makes easy reading.
    – River Li Oct 24 '21 at 13:59
  • $m\left(a\right)+m\left(2-a+0.5\left(a-1\right)^{2}\right)=k(a)=\cdots$, I think it is not nice to put the definition of $k(a)$ here. Perhaps it is better to "Let $k(a) = \cdots$". – River Li Oct 24 '21 at 14:00
  • Actually, $$ h\left(2-a+0.5\left(a-1\right)^{2}\right) = m(a)+m\left(2-a+0.5\left(a-1\right)^{2}\right) = k(a).$$ It is not necessary to use all $h, m, k$. For example, directly $[h\left(2-a+0.5\left(a-1\right)^{2}\right)]''' < 0$ without introducing $m(a), k(a)$, or let $k(a) = h\left(2-a+0.5\left(a-1\right)^{2}\right)$, then $k'''(a) < 0$. – River Li Oct 25 '21 at 00:53
  • By $f'(2-a+0.5\left(a-1\right)^{2})$, do you mean $\frac{\partial f}{\partial x}\Big\vert_{x = 2-a+0.5\left(a-1\right)^{2}}$, or $[f(2-a+0.5\left(a-1\right)^{2})]'$? – River Li Oct 25 '21 at 01:12
  • I mean $b(x)=f'(x)$ then $b(2-a+0.5(a-1)^2)=f'(2-a+0.5(x-1)^2)$ – Barackouda Oct 25 '21 at 11:24
  • I think, you may write: Let $F(a) = \frac{\partial f}{\partial x}\Big\vert_{x = 2-a+0.5\left(a-1\right)^{2}}$ (something like that). Then $F'(a) = \cdots$ (replace $(f'(2-a+0.5\left(a-1\right)^{2}))'$). – River Li Oct 25 '21 at 11:29